LSAT and Law School Admissions Forum

Get expert LSAT preparation and law school admissions advice from PowerScore Test Preparation.

 Administrator
PowerScore Staff
  • PowerScore Staff
  • Posts: 8916
  • Joined: Feb 02, 2011
|
#36512
Complete Question Explanation

Assumption. The correct answer choice is (A)

This stimulus starts as many LSAT questions do—with a prevailing view to be disputed. In this
case, the prevailing view is that the difference in wages between highest and lowest will eventually
become a source of social confl ict. The sociologist disputes this claim, concluding that the difference
will have an opposite effect, because companies will be able to hire freely. He claims that social
friction does not arise from high wage differences, but rather from static or slowly changing wages.
  • Premise: Social friction arises from static or slow changing wages.

    Conclusion: The ability to hire freely will reduce social friction.
In order for this conclusion to be properly drawn, the author must believe that the ability to hire
freely must somehow make static or slow changing wages less likely.

The question stem asks for the assumption required by the sociologist’s argument; this is a Supporter
Assumption question, and the correct answer choice should provide some link between the ability to
hire freely and the absence of static or slow changing wages—for the author’s conclusion to hold, it
must be the case that the ability to hire freely must allow companies to avoid static or slow changing
wages.

Answer choice (A): This is the correct answer choice, as it provides the link between the rogue
elements referenced above: When companies can hire freely, wage levels tend not to be static or slow
changing:
  • Premise: Social friction arises from static or slow changing wages.

    Assumption: There tend not to be slow or static changes to wages when companies are able
    to hire freely.


    Conclusion: The ability to hire freely will reduce social friction.
Once we make this Supporter Assumption explicit, we can see that the argument flows logically.

Answer choice (B): The conclusion in the stimulus does not concern reactions to wage change
expectations, but rather the relationship between static or slowly changing wages, social friction, and
companies’ ability to hire freely. Since the sociologist’s conclusion makes no distinction regarding
how people react to disparities in income, and this choice makes no reference to the unlinked
elements of the author’s argument, this answer choice is incorrect.

Answer choice (C): Expansion of business operations is completely irrelevant to the sociologist’s
conclusion, and this answer does not tie together the prephrased elements listed above, so this
answer choice is incorrect.

Answer choice (D): It may be the case that a company’s ability to respond swiftly to change always
benefits workers, but since this choice does not provide the necessary link between the rogue
elements referenced above, this is not the Supporter Assumption we are seeking.

Answer choice (E): If well-paid workers become dissatisfied if their wages never change, this
may help to support or explain the author’s assertion that static wages lead to social friction, but
this answer choice does not link the premise about static wages causing social friction with the
conclusion that when companies can hire freely, social friction is reduced.
 avengingangel
  • Posts: 275
  • Joined: Jun 14, 2016
|
#28672
Can you please show me the full diagram of this argument?

I'm not sure if you diagram "be able to hire freely in response to changing conditions" as one variable (being the effect of large wage difference cause), OR if you break that up into a conditional/causal relationship of it's own (&, which would be on each side of the relationship??) ?? When I diagrammed this out, there were no pieces that were missing (unstated) so I didn't know how to approach this Assumption question. I only chose (A) because it was the only one that linked in the new (rogue) info of static or slow changing wage levels in the answer. And, is the conclusion "high differential should have an opposite effect" ?that's that I think it is now, but I originally thought it was the last sentence, that "Social friction arises from wage levels that are static or slow changing," but that definitely seems like a premise. So many questions. Please help!!
 Nikki Siclunov
PowerScore Staff
  • PowerScore Staff
  • Posts: 1362
  • Joined: Aug 02, 2011
|
#28865
Angel,

I just wrote a response on the topic of putting too much emphasis on the distinction between Supporter and Defender assumptions, particularly as it pertains to our handling of "new information" in the conclusion. Check it out, and be wary of such "tricks": they work, until they don't :) Nothing can replace a solid conceptual understanding of why an argument like this is not logically valid.

The argument is structured as follows:

Premise 1: Wage inequality will cause companies to hire freely in response to changing conditions.
  • ..... ..... ..... Cause ..... ..... ..... Effect

    ..... ..... Wage inequality :arrow: Hire freely
Premise 2: Static/low wages cause social friction
  • ..... ..... ..... Cause ..... ..... ..... Effect

    ..... ..... Static/low wages :arrow: Social friction
Conclusion: The contention of the sociologist's colleagues is wrong, i.e. wage inequality need not become a source of social friction, and should actually have the opposite effect.
  • ..... ..... ..... Cause ..... ..... ..... Effect

    ..... ..... Wage inequality :arrow: NO social friction
Do you see the gap in the sociologist's reasoning? The fact that companies will be able to hire freely does not, by itself, preclude the possibility of social friction. Who is to say that hiring freely won't keep wages low? The author is assuming that when companies are free to hire in response to changing conditions, wages will magically go up:

Assumption:
  • ..... ..... ..... Cause ..... ..... ..... Effect

    ..... ..... Hire freely :arrow: NO static/low wages
We can prove that this is the assumption by negating answer choice (A). What if the following proposition is true:
Logical opposite of answer choice (A), simplified: When companies hire freely, wages levels tend to remain static.
If true, this immediately shows that the sociologist's recipe for avoiding social friction isn't going to work out. After all, he himself acknowledged that social friction arises from wage levels that are static.

Hope this clears it up! Let me know if you have any follow-up questions.

Thanks!

Get the most out of your LSAT Prep Plus subscription.

Analyze and track your performance with our Testing and Analytics Package.